Validity of predicate logic arguments

A.H.S

New member
Joined
Apr 19, 2021
Messages
2
I got stuck in Predicate logic practice exercises. Please guide how can I check that the below arguments/formula are valid or not?

1618846915179.png




1618846925627.png
 

Attachments

  • 1.PNG
    1.PNG
    25 KB · Views: 8
Again. Here is my attempt.

For part (a)

[MATH] \forall y \forall z (R(y,z) \rightarrow \neg P(z)) \text{, }P(a) \vDash \exists z \forall y(\neg R(y,z))[/MATH]Valid

[MATH] \text {(1) } \forall y \forall z (R(y,z) \rightarrow \neg P(z)) \hspace7ex \text{Premise}\\ \text {(2) } \forall y \forall z (P(z) \rightarrow \neg R(y,z)) \\ \text {(3) } \forall y (P(a) \rightarrow \neg R(y,a)) \hspace9ex \text{Universal Specification}\\ \text {(4) }P(a) \quad \quad \hspace20ex \text{Premise}\\ \text {(5) }\forall y (P(a) \rightarrow \neg R(y,a)) \land P(a) \hspace2ex \text{(3), (4)}\\ \text {(6) }\forall y ((P(a) \rightarrow \neg R(y,a)) \land P(a))\\ \text {(7) }\forall y ( \neg R(y,a)) \hspace17ex \text{MP (4), (6)}\\ \text {(8) }\exists z \forall y(\neg R(y,z)) \hspace15ex \text{Existential Generalisation}[/MATH][MATH]\text{_______________________________________}\\[10pt] \vDash \forall y \forall z (R(y,z) \rightarrow \neg P(z)) \land P(a) \rightarrow \exists z \forall y(\neg R(y,z))\\ \forall y \forall z (R(y,z) \rightarrow \neg P(z)) \text{, }P(a) \vDash \exists z \forall y(\neg R(y,z))[/MATH]
For part (b)
[MATH] \vDash \exists xP(x) \land Q(y) \rightarrow \exists z(P(z) \land Q(z))[/MATH]Not valid
Just because there exists a [MATH]x[/MATH]: P(x) true and that Q(y) true, doesn't mean there exists an [MATH]x[/MATH]: both P(x) true and Q(x) true.
[L.h.s [MATH]\rightarrow \exists x: P(x)[/MATH], so [MATH] P(a)[/MATH] true for some [MATH]a[/MATH].
L.h.s. [MATH]\rightarrow Q(y)[/MATH] true.
We know nothing about Q(a).
So we cannot say that [MATH]P(a) \land Q(a[/MATH]) true.
So we cannot conclude that [MATH]\exists z: P(z) \land Q(z)[/MATH] ].

Suppose that P(y) false, Q(y) true, and [MATH]\exists x≠y: P(x)[/MATH] true.
Consider a case where [MATH]\forall x P(x)= \neg Q(x)[/MATH] (this is not in conflict with the previous line),
then [MATH]\exists x P(x) \land Q(y)[/MATH], so L.h.s is true and
[MATH]\forall P(x)=\neg Q(x) \text{ so } \neg \exists x: P(x) \land Q(x)[/MATH], so R.h.s. is false.
[MATH]\text{_______________________________________}\\[10pt] \exists x P(x) \land Q(y) \not \vDash \exists z(P(z) \land Q(z))\\ \not \vDash \exists xP(x) \land Q(y) \rightarrow \exists z(P(z) \land Q(z))[/MATH]
 
Top